Find sin 0. Please and thank you

Find Sin 0. Please And Thank You

Answers

Answer 1

Answer:

the correct answer is C....I hope


Related Questions

P(x) is a polynomial. here are a few values of p(x).
P(-5) = - 2
P(-3) = 6
P(3) = 7
P(5) = -1
What is the remainder when P(x) is divided by (x+5)?
What is the remainder when P(x) is divided by (x-3)?

Answers

Given:

Values of a polynomial P(x).

[tex]P(-5)=-2[/tex]

[tex]P(-3)=6[/tex]

[tex]P(3)=7[/tex]

[tex]P(5)=-1[/tex]

To find:

The remainder when P(x) is divided by (x+5).

The remainder when P(x) is divided by (x-3).

Solution:

If a polynomial P(x) is divided by (x-a), then the remainder is P(a).

If the polynomial P(x) is divided by (x+5), then the remainder is P(-5).

[tex]P(-5)=-2[/tex]

Therefore, the remainder is -2 when P(x) is divided by (x+5).

If the polynomial P(x) is divided by (x-3), then the remainder is P(3).

[tex]P(3)=7[/tex]

Therefore, the remainder is 7 when P(x) is divided by (x-3).

On a coordinate plane, a line goes through (negative 3, negative 4) and (3, 0).
What are the necessary criteria for a line to be perpendicular to the given line and have the same y-intercept?

The slope is Negative three-halves and contains the point (0, 2).
The slope is Negative two-thirds and contains the point (0, −2).
The slope is Three-halves and contains the point (0, 2).
The slope is Negative three-halves and contains the point (0, −2

Answers

Answer:

The slope is Negative three-halves and contains the point (0, 2).

Step-by-step explanation:

(-3,-4)(3,0)

M= -4/-6 = 2/3

⊥M = -3/2

0 = 2/3(3) + B

B=2

are cans of bottles packaged at a faster rate

Answers

Answer:

yes.. they are

Step-by-step explanation:

because

Please help me with this one I seriously suck at math

Answers

Answer:

194

Step-by-step explanation:

Rectangles :

7 x 10 = 70

6 x 10 = 60

4 x 10 = 40

Triangles

[tex]\frac{6x4}{2}[/tex] = 12   (two of them = 24)

70 + 60 + 40 + 24 = 194

Hermes says that the opposite of 9 is 9 since
each number is 9 units from zero on the number line. Is Hermes correct?
Explain why or why not

Answers

Answer: Hermès is incorrect. The opposite of 9 is -9.


Explanation: In this question, Hermès gives an incorrect answer, claiming that the opposite of 9 is also 9. To find the correct answer, let’s first look at his reasoning. He was right in one thing- to find the opposite of a number, we need to find one that shares the same distance from zero. However, he was wrong in believing that the number had to be exactly the same. To find the opposite number, find the same distance from zero in the opposite direction. In this case, the distance from zero is 9. Go in the opposite direction from zero, 9 times. This would give you an answer of -9. Because of this, the opposite of 9 is -9.


Hope this helps! Comment below for more questions.

Hermes is incorrect. The opposite of 9 is -9 if Hermes says that the opposite of 9 is 9 since each number is 9 units from zero on the number line.

What is a number line?

It is defined as the representation of the numbers on a straight line that goes infinitely on both sides.

To find the same distance away from zero in the opposite direction to determine the opposite number.

The distance from zero in this situation is 9. nine times in the opposite direction from zero.

Thus, Hermes is incorrect. The opposite of 9 is -9 if Hermes says that the opposite of 9 is 9 since each number is 9 units from zero on the number line.

Learn more about the number line here:

brainly.com/question/13189025

#SPJ2

log8-log4 ÷ log4-log2=





Answers

The answer is log(4)-1

if f(x) = 2x/7 +4, which of the following is the inverse of f(x)?

Answers

Answer:

The answer is C

Step-by-step explanation:

Switch x and y

[tex]x = \frac{2y}{7} + 4[/tex]

Solve for y

[tex]y = \frac{7(x - 4)}{2} [/tex]

3 questions only pls help gota finish tofday

Answers

5a) 3x3x13

b) 3x7x7

c)11x5x2x2

d)3x3x3x5x2

6a)2x2x2x131

7a)5x5x2x2

b) 3x2x17

c)5x2x2x2x2x2x2

d)5x5x5x3

e) 2x2x103

f)5x3x137

g) 2x2x2x2x2x2x2x2x2

h)5x5x7x7x3

sue has 18 pieces of candy
tony has 18 pieces of candy
sue then gives some to tony
sue then eats five of hers
tony eats half of his
write the expressions for the number of pieces candy sue and tony now have?

Answers

Answer:

Sue candy = 13 - x

Tony candy = 9 + 1/2x

Step-by-step explanation:

Sue candy = 18

Tony candy = 18

Let x = some candy gives to tony

Sue candy = 18 - x

Tony candy = 18 + x

sue then eats five of hers

Sue candy = 18 - x - 5

= 13 - x

tony eats half of his

Tony candy = 1/2(18 + x)

= 18/2 + x/2

= 9 + 1/2x

Expressions for the number of pieces candy sue and tony now have:

Sue candy = 13 - x

Tony candy = 9 + 1/2x

One leg of a right triangle is 7 inches longer than the other leg, and the hypotenuse is 35 inches. Find the lengths of the legs of the triangle.

Answers

Answer: 21, 28

Step-by-step explanation:

Side #1 = xSide #2 = x + 7Hypotenuse = 35

Use the Pythagorean Theorem [tex]a^{2}+b^{2}=c^{2}[/tex]:

a = xb = x + 7c = 35

Substitute in the values & solve:

[tex]x^{2}+(x+7)^{2}=35^{2}\\x^{2}+x^{2}+14x+49=1225\\2x^{2}+14x+49-1225=0\\2x^{2}+14x-1176=0\\2(x^{2}+7x-588)=0\\2(x + 28)(x - 21)=0\\x_{1}=-28, x_{2}=21[/tex]

-28 is not a possible solution since you can't have negative inches...

a = x = 21b = x + 7 = 21 + 7 = 28c = 35

Please help. I don’t understand I rlly appreciate it if you help!

Answers

Hi there!

We know that the angles ∠B and ∠A are supplementary because they are both interior angles. Therefore:

180 = ∠A + ∠B

Rewrite:

180 = (6x - 48) + (4x + 38)

Combine like terms:

180 = 10x - 10

Solve for x:

190 = 10x

x = 19

Solve for ∠B by plugging in this value of x:

∠B = 4(19) + 38

∠B = 76 + 38

∠B = 114

What is the slope of the line? What is the y-intercept of the line? y = -3x + 4

Answers

Answer:

slope= -3/1

y-intercept= 4

Answer:

m = -3

y intercept = 4

Step-by-step explanation:

The given equation of the line is ,

[tex]\implies y = -3x+4[/tex]

We know that the Standard equation of Slope Intercept Form of the line is,

[tex]\implies y = mx + c[/tex]

Where ,

m is slope c is y intercept

On comparing to the Standard form of the line we get ,

[tex]\implies Slope = -3 [/tex]

[tex]\implies y - intercept= 4 [/tex]

Twenty of the 50 digital video recorders (DVRS) in an inventory are known to be detective What is the probability that a randomly selected item is defective?​

Answers

Answer:

3/5

Step-by-step explanation:

Remember that odds for plus odds against must be 1. Therefore, 20 out of 50 of the DVRs are defective, so 30 out of 50 would work.

30/50

simplifies into 3/5 so that would be the probability of you picking out a DVR that works, and the probability of picking one that does not work would be the leftover 2/5 . Or, if you're answering with a percent, you would have a 60% chance of picking a working DVR.

I hope this helps, I know the explanation isn't really clear, but I can't really think of another way to explain it.

Please Help!!!










Find a value for C that will give the following system no solution:
3x-2y=3
6x+cy=4

Answers

Answer:

c = - 4

Step-by-step explanation:

Nothing else matters except that the number associated with the y value in equation one = 1 and the number in front of x is known in equation 1. Then we'll get around to looking at c

Add 2y to both sides of equation 1 (the top equation)

3x - 2y + 2y = 3 + 2y

3x = 2y + 3        

Now divide both sides of the equation by 2

3x/2 = y + 3/2

What's in front of x? It is (3/2)

So divide both sides of 6x + cy = 4       by c

6x/c + y = 4/c

Now put 6x/c on the right hand side so y is by itself.

y = -6x/c + 4/c

What do you have to do now?

You must equate -6x/c = 3/2x

Why?

The slopes have to be the same so the lines are parallel and never cross. That will give no solutions.

-6x/c = 3x / 2     Divide both sides by x

-6/c = 3/2           Cross multiply

3c = - 6 * 2        Combine

3c = - 12             Divide by 3

c = - 12/3

c = - 4

A number ending in ___ is never a perfect square. ​

Answers

Answer:

2, 3, 7 or 8

Step-by-step explanation:

what is the length of segment DC? no links.

Answers

Answer:

33 units

Step-by-step explanation:

AB = BC

2x + 7 = 23

2x = 16

x = 8

DC = DA

DC = 4x + 1

DC = 4(8) + 1

DC = 33

t=29pi/6
1. find the reference number
2. find the point on the unit circle
3. 6 trig functiond

Answers

the correct answer is C....I hope

In In 5x + In In (x - 1) = 2

Answers

Answer:

exact form: x=-1/2

decimal form: x=-0.5

What is the equation of the line??

I NEED ANSWERS NOW PLEASEEEEE

for 20 points pls people help me out

Answers

Answer:

x = 0

Step-by-step explanation:

This is a vertical line, in fact the y- axis

The equation of a vertical line is

x = c

where c is the value of the x- coordinates the line passes through

All of the x- coordinates on the y- axis are zero, then

x = 0 ← equation of line

Identify a horizontal or vertical stretch or compression of the function by observing the equation of the function .

Answers

Bdbxbcjncnxndnsnns Identify a horizontal or vertical stretch or compression of the function by observing the equation of the function .hdbdbebqb

Solve for x. Round to the nearest tenth, if necessary.

Answers

Answer:

11.3

Step-by-step explanation:

first we find angle F.

remember, all angles in a triangle always sum up to 180 degrees.

so,

F = 180 - 90 - 61 = 29 degrees

now we use the law of sines.

EF/sin(D) = ED/sin(F) = DF/sin(E)

DF = x

sin(E) = sin(90) = 1

5.5/sin(29) = x/1 = x

x = 11.3

one more question
(-8)+___=-17

Answers

Answer:

-9

Step-by-step explanation:

Answer:

-9

Step-by-step explanation:

-8 +___= - 17

___=-17 +8

___=-9

=-9

Which point is in the solution set of this system inequalities?

A. (0,0)

B. None of these

C. (5,1)

D. (3,7)

Answers

Answer:

B

Step-by-step explanation:

To find which ordered pairs are solutions to the inequalities we can simply plug in the x and y values of the ordered pairs into the inequalities and if the equation is true for both inequalities then the ordered pair is a solution to the inequalities.

For (0,0)

x = 0

y = 0

y > x + 5

Substitute 0 for y and x

0 > 0 + 5

Simplify right side

0 > 5

The inequality is not true as 5 is greater than 0, not less than. So immediately we can eliminate answer choice A.

For (5,1).

x = 5

y = 1

y > x + 5

Substitute 5 for x and 1 for y

1 > 5 + 5

Simplify right side

1 > 10

Again, the equation is not true as 1 is not greater than 10. This means that c cannot be the answer

For (3,7)

x = 3

y = 7

y > x + 5

Substitute 3 for x and y for 7

7 > 3 + 5

Simplify right side

7 > 8

7 is not greater than 8 meaning that (3,7) cannot be a solution to the inequalities

None of the ordered pairs created true equations hence the answer is B

estimate the solution to the following system of equations by graphing.
3x+5y=14
6x-4y=9

A. ( 4/3,5/2)
B. ( 7/3, -7/2 )
C. ( -5/2, -7/2)
D. (5/2, 4/3) ​

Answers

Answer:

(101/42, 19/14)

Step-by-step explanation:

Given the following equations

3x+5y=14 ..... 1 * 2

6x-4y=9 ... 2 * 1

__________________

6x+10y=28

6x-4y=9  

Subtract

10y+4y = 28 - 9

14y = 19

y = 19/14

Get the value of x

Since 3x+5y = 14

3x+5(19/14) = 14

3x + 95/14 = 14

3x = 14 - 95/14

3x = 101/14

42x = 101

x = 101/42

Hence the solution is (101/42, 19/14)

Recipe ingredients remain jn a constant ratio no matter how many serving are prepared. Which table shows a possible ratio table for ingredients C and Y for the given number of servings

Answers

Answer:

The last table (the bottom one)

Step-by-step explanation:

The ingredients having the same ratio means that, for every number of servings, we should have:

Y/X = constant.

So, for the first table when we have 1 serving, the quotient is:

Y/X = 2/1 = 2

when we have two servings:

Y/X = 3/2 = 1.5

The ratios are different.

Then this is not the correct option.

For the second table, when we have 1 serving the ratio is:

Y/X = 2/1 = 2

when we have two servings:

Y/X = 4/2 = 2

when we have 3 servings:

Y/X = 8/3 = 2.66

This is not the correct option.

For the third table:

1 serving:

Y/X = 2/1 = 2

2 sevings

Y/X = 3/2 = 1.5

This is not the correct option.

fourth table:

1 serving:

Y/X = 2/1 = 2

2 servings

Y/X = 4/2 = 2

3 servings

Y/X = 8/4 = 2

Here we can see that the ratio is always the same, then the ratio remains constant.

This is the table that shows a possible ratio for ingredients X and Y,

Drag the operator to the correct location on the image.
Which operation results in a binomial?

Answers

The correct answer is to drag The Plus sign (+)

What is an Operator?

This has to do with the use of symbols to denote mathematical equations such as addition, subtraction, etc.

Hence, we can see that the correct position to put the operator on the image to result in a binomial is to drag the plus sign (+) so that the equations can be solved,.

Read more about operators here:

https://brainly.com/question/25974538

#SPJ2

Answer:.

Step-by-step explanation:

8^5 = 2^2m+3


Solve m

Answers

Answer:

[tex]m=6[/tex]

Step-by-step explanation:

Exponent properties:

We can use exponent property [tex]a^{b^c}=a^{(b\cdot c)}[/tex] to solve this problem.

Rewrite [tex]8[/tex] as [tex]2^3[/tex], then apply exponent property [tex]a^{b^c}=a^{(b\cdot c)}[/tex] to simplify:

[tex]2^{3^5}=2^{2m+3},\\2^{15}=2^{2m+3}[/tex]

If [tex]a^b=a^c[/tex], then [tex]b=c[/tex], because of log property [tex]\log a^b=b\log a[/tex]. Using this log property, you can take the log of both sides and divide by [tex]\log a[/tex] to get [tex]b=c[/tex]

Therefore, we have:

[tex]15=2m+3[/tex]

Subtract 3 from both sides:

[tex]12=2m[/tex]

Divide both sides by 6:

[tex]m=\frac{12}{2}=\boxed{6}[/tex]

Alternative:

Given [tex]8^5=2^{2m+3}[/tex], to move the exponent down, we'll use log properties.

Start by simplifying:

[tex]\log 32,768=2^{2m+3}[/tex]

Take the log of both sides, then use log property [tex]\log a^b=b\log a[/tex] to move the exponent down:

[tex]\log(32,768)=\log 2^{2m+3},\\\log (32,768)=(2m+3)\log 2[/tex]

Divide both sides by [tex]\log2[/tex]:

[tex]2m+3=\frac{\log (32,768)}{\log(2)}[/tex]

Subtract 3 from both sides:

[tex]2m=\frac{\log (32,768)}{\log(2)}-3[/tex]

Divide both sides by 2:

[tex]m=\frac{\log (32,768)}{2\log(2)}-\frac{3}{2}=\boxed{6}[/tex]

WILL GIVE BRAINLIEST AND 50 POINTS!
Determine the more basic function that has been shifted, reflected, stretched, or compressed.
m(x)= 2√x-5 -2

Answers

Answer: The function M if derived from the integral of the derivative of the function. This is valid by the fundamental theorem of calculus.

M(x)=43x32−7x+C

Step-by-step explanation: not 100 about this but it's what I got

Someone please help me ASAP

Answers

Step-by-step explanation:

a vector multiplied by a scalar is equal to it's image. The expression above gives an equation and after solving, it gives you the image

someone help me for this algebra task please

Answers

Answer:

200

Step-by-step explanation:

Substitute 15 for y

[tex] \frac{1}{5} x - \frac{2}{3} (15) = 30[/tex]

[tex] \frac{1}{5} x - 10 = 30[/tex]

[tex] \frac{1}{5} x = 40[/tex]

[tex]x = 200[/tex]

Other Questions
The following measurements are taken on particular junction diodes for which V is the terminal voltage and I is the diode current. For each diode, estimate values of Is and the terminal voltage at 10% of the measured current.(a) V = 0.700 V at I = 1.00 A.(b) V = 0.650 V at I = 1.00 mA.(c) V = 0.650 V at I = 10 mu A.(d) V = 0.700V at I = 100 mA. Ice and water constitute a system:A Chemically heterogeneous and physically heterogeneous B Chemically homogeneous and physically heterogeneous C Chemically homogeneous and physically homogeneous D Chemically heterogeneous and physically homogeneous E None of the above help plzzzzzzzzzzzzzzzzzzzzzzzzzzz No trolling please. I will mark brainlist Atolls are geological formations made up of : What is the range of this set of heights in centimeters? {140, 166, 132, 165, 152, 168, 181, 158, 173, 171, 180, 182, 163, 177, 180, 142, 147, 149, 178} 38 41 46 50 Why does the main character of Into the wild went on his epic journey to Alaska? Solve the following system. The solution set {( , ), ( , )}. Sonia hated doing chores. Furthermore, she was bad atthem. She and her sister needed to fix up the house beforeselling it, however, so there was no getting around thework. The porch needed to be painted, the fridge needed tobe replaced, and the carpet needed to be shampooed.Suddenly, Sonia had an idea. All of these tasks would costmoney. Maybe she could work out a deal with her sister topay for everything - the paint, the new fridge, and thecarpet shampoo - if her sister did all the work.Which option best describes the rhetorical situation in this scenario?O A. Audience: Potential buyers of the housePurpose: To sell the houseContext: Passing the house on to new ownersOB. Audience: A general audiencePurpose: To pay for suppliesContext: The houseO C. Audience: Sonia's sisterPurpose: To get out of doing choresContext: Fixing up the house before selling itO D. Audience: SoniaPurpose: To fix up the new houseContext: Improving the condition of the house i need help please answer this(FILIPINO AKO) Which of the following is not an example of biasA. Pro hi FRUCTOSE corn syrup ADVERTISEMENTS paid for by corn farmers B. anti-smoking articles written by the national health ASSOCIATIONC. anti-by four articles written by a journalist working for it Oil companyD Pro ADHD medicine article written by PHARMACEUTICAL Eight months ago, you purchased 400 shares of Winston, Inc. stock at a price of $56.90 a share. To date the company has paid quarterly dividends of $.55 a share twice. Today, you sold all of your shares for $49.40 a share. What is your total percentage return on this investment Gershwin Wallcovering Inc. shipped the wrong shade of paint to a customer. The customer agreed to keep the paint upon being offered a 15% price reduction. The price reduction is an example of a: TIME REMAINING56:39Which statement is true about inorganic compounds?They are not made of atoms.They contain carbon.They are not made from living things.They are all elements. In steps 1, 2, and 3, you will use a Punnett square to predict Mi quan h gia i tng v lp1. Lp c trc, i tng c sau, i tng l thnh phn ca lp2. Lp l tp hp cc i tng c cng kiu d liu nhng khc nhau v cc phng thc3. i tng l th hin ca lp, mt lp c nhiu i tng cng thnh phn cu trc4. i tng i din cho lp, mi lp ch c mt i tng Help ! ASAP please and thank you !! Wavelength varies inversely with frequency. Let k be the product of wavelength and frequency. Complete the table using the inverse variation relationship. x + 9x + 5 = 35 . Find value of x. Any help is much appreciated